You are on page 1of 123

Complete Solutions

to the Physics GRE

Exam #0177

Taylor Faucett
Senior Editor: Taylor Faucett

Editor-in-Chief: Taylor Faucett

Associate Editor: Taylor Faucett

Editorial Assistant: Taylor Faucett

Art Studio: Taylor Faucett

Art Director: Taylor Faucett

Cover Design: Taylor Faucett

Cover Image: Mandelbrot inkblot test

BiBTeX:

@Book{Faucett0177,
author = {Taylor Faucett},
series = {Complete Solutions to the Physics GRE},
title = {Exam #0177},
pages = {36--38},
year = {2010},
edition = {first},
}

c 2010 Taylor Faucett


The “Complete Solutions to the Physics GRE” series has been produced strictly for
educational and non-profit purposes. All information contained within this document
may be copied and reproduced provided that these intentions are not violated.
Contents

1 Physics GRE Solutions 3


1.1 PGRE0177 #1 . . . . . . . . . . . . . . . . . . . . . . . . . . . . . . . . . . . . . . . 3
1.2 PGRE0177 #2 . . . . . . . . . . . . . . . . . . . . . . . . . . . . . . . . . . . . . . . 4
1.3 PGRE0177 #3 . . . . . . . . . . . . . . . . . . . . . . . . . . . . . . . . . . . . . . . 6
1.4 PGRE0177 #4 . . . . . . . . . . . . . . . . . . . . . . . . . . . . . . . . . . . . . . . 7
1.5 PGRE0177 #5 . . . . . . . . . . . . . . . . . . . . . . . . . . . . . . . . . . . . . . . 8
1.6 PGRE0177 #6 . . . . . . . . . . . . . . . . . . . . . . . . . . . . . . . . . . . . . . . 9
1.7 PGRE0177 #7 . . . . . . . . . . . . . . . . . . . . . . . . . . . . . . . . . . . . . . . 10
1.8 PGRE0177 #8 . . . . . . . . . . . . . . . . . . . . . . . . . . . . . . . . . . . . . . . 11
1.9 PGRE0177 #9 . . . . . . . . . . . . . . . . . . . . . . . . . . . . . . . . . . . . . . . 12
1.10 PGRE0177 #10 . . . . . . . . . . . . . . . . . . . . . . . . . . . . . . . . . . . . . . . 13
1.11 PGRE0177 #11 . . . . . . . . . . . . . . . . . . . . . . . . . . . . . . . . . . . . . . . 14
1.12 PGRE0177 #12 . . . . . . . . . . . . . . . . . . . . . . . . . . . . . . . . . . . . . . . 15
1.13 PGRE0177 #13 . . . . . . . . . . . . . . . . . . . . . . . . . . . . . . . . . . . . . . . 17
1.14 PGRE0177 #14 . . . . . . . . . . . . . . . . . . . . . . . . . . . . . . . . . . . . . . . 18
1.15 PGRE0177 #15 . . . . . . . . . . . . . . . . . . . . . . . . . . . . . . . . . . . . . . . 19
1.16 PGRE0177 #16 . . . . . . . . . . . . . . . . . . . . . . . . . . . . . . . . . . . . . . . 21
1.17 PGRE0177 #17 . . . . . . . . . . . . . . . . . . . . . . . . . . . . . . . . . . . . . . . 22
1.18 PGRE0177 #18 . . . . . . . . . . . . . . . . . . . . . . . . . . . . . . . . . . . . . . . 23
1.19 PGRE0177 #19 . . . . . . . . . . . . . . . . . . . . . . . . . . . . . . . . . . . . . . . 24
1.20 PGRE0177 #20 . . . . . . . . . . . . . . . . . . . . . . . . . . . . . . . . . . . . . . . 25
1.21 PGRE0177 #21 . . . . . . . . . . . . . . . . . . . . . . . . . . . . . . . . . . . . . . . 26
1.22 PGRE0177 #22 . . . . . . . . . . . . . . . . . . . . . . . . . . . . . . . . . . . . . . . 28
1.23 PGRE0177 #23 . . . . . . . . . . . . . . . . . . . . . . . . . . . . . . . . . . . . . . . 29
1.24 PGRE0177 #24 . . . . . . . . . . . . . . . . . . . . . . . . . . . . . . . . . . . . . . . 30
1.25 PGRE0177 #25 . . . . . . . . . . . . . . . . . . . . . . . . . . . . . . . . . . . . . . . 31
1.26 PGRE0177 #26 . . . . . . . . . . . . . . . . . . . . . . . . . . . . . . . . . . . . . . . 33
1.27 PGRE0177 #27 . . . . . . . . . . . . . . . . . . . . . . . . . . . . . . . . . . . . . . . 35
1.28 PGRE0177 #28 . . . . . . . . . . . . . . . . . . . . . . . . . . . . . . . . . . . . . . . 36
1.29 PGRE0177 #29 . . . . . . . . . . . . . . . . . . . . . . . . . . . . . . . . . . . . . . . 37
1.30 PGRE0177 #30 . . . . . . . . . . . . . . . . . . . . . . . . . . . . . . . . . . . . . . . 38
1.31 PGRE0177 #31 . . . . . . . . . . . . . . . . . . . . . . . . . . . . . . . . . . . . . . . 39
1.32 PGRE0177 #32 . . . . . . . . . . . . . . . . . . . . . . . . . . . . . . . . . . . . . . . 40
1.33 PGRE0177 #33 . . . . . . . . . . . . . . . . . . . . . . . . . . . . . . . . . . . . . . . 41
1.34 PGRE0177 #34 . . . . . . . . . . . . . . . . . . . . . . . . . . . . . . . . . . . . . . . 42
1.35 PGRE0177 #35 . . . . . . . . . . . . . . . . . . . . . . . . . . . . . . . . . . . . . . . 43
1.36 PGRE0177 #36 . . . . . . . . . . . . . . . . . . . . . . . . . . . . . . . . . . . . . . . 44

3
CONTENTS CONTENTS

1.37 PGRE0177 #37 . . . . . . . . . . . . . . . . . . . . . . . . . . . . . . . . . . . . . . . 45


1.38 PGRE0177 #38 . . . . . . . . . . . . . . . . . . . . . . . . . . . . . . . . . . . . . . . 46
1.39 PGRE0177 #39 . . . . . . . . . . . . . . . . . . . . . . . . . . . . . . . . . . . . . . . 47
1.40 PGRE0177 #40 . . . . . . . . . . . . . . . . . . . . . . . . . . . . . . . . . . . . . . . 48
1.41 PGRE0177 #41 . . . . . . . . . . . . . . . . . . . . . . . . . . . . . . . . . . . . . . . 49
1.42 PGRE0177 #42 . . . . . . . . . . . . . . . . . . . . . . . . . . . . . . . . . . . . . . . 50
1.43 PGRE0177 #43 . . . . . . . . . . . . . . . . . . . . . . . . . . . . . . . . . . . . . . . 51
1.44 PGRE0177 #44 . . . . . . . . . . . . . . . . . . . . . . . . . . . . . . . . . . . . . . . 52
1.45 PGRE0177 #45 . . . . . . . . . . . . . . . . . . . . . . . . . . . . . . . . . . . . . . . 53
1.46 PGRE0177 #46 . . . . . . . . . . . . . . . . . . . . . . . . . . . . . . . . . . . . . . . 54
1.47 PGRE0177 #47 . . . . . . . . . . . . . . . . . . . . . . . . . . . . . . . . . . . . . . . 55
1.48 PGRE0177 #48 . . . . . . . . . . . . . . . . . . . . . . . . . . . . . . . . . . . . . . . 56
1.49 PGRE0177 #49 . . . . . . . . . . . . . . . . . . . . . . . . . . . . . . . . . . . . . . . 57
1.50 PGRE0177 #50 . . . . . . . . . . . . . . . . . . . . . . . . . . . . . . . . . . . . . . . 58
1.51 PGRE0177 #51 . . . . . . . . . . . . . . . . . . . . . . . . . . . . . . . . . . . . . . . 60
1.52 PGRE0177 #52 . . . . . . . . . . . . . . . . . . . . . . . . . . . . . . . . . . . . . . . 62
1.53 PGRE0177 #53 . . . . . . . . . . . . . . . . . . . . . . . . . . . . . . . . . . . . . . . 63
1.54 PGRE0177 #54 . . . . . . . . . . . . . . . . . . . . . . . . . . . . . . . . . . . . . . . 65
1.55 PGRE0177 #55 . . . . . . . . . . . . . . . . . . . . . . . . . . . . . . . . . . . . . . . 66
1.56 PGRE0177 #56 . . . . . . . . . . . . . . . . . . . . . . . . . . . . . . . . . . . . . . . 67
1.57 PGRE0177 #57 . . . . . . . . . . . . . . . . . . . . . . . . . . . . . . . . . . . . . . . 68
1.58 PGRE0177 #58 . . . . . . . . . . . . . . . . . . . . . . . . . . . . . . . . . . . . . . . 69
1.59 PGRE0177 #59 . . . . . . . . . . . . . . . . . . . . . . . . . . . . . . . . . . . . . . . 70
1.60 PGRE0177 #60 . . . . . . . . . . . . . . . . . . . . . . . . . . . . . . . . . . . . . . . 71
1.61 PGRE0177 #61 . . . . . . . . . . . . . . . . . . . . . . . . . . . . . . . . . . . . . . . 72
1.62 PGRE0177 #62 . . . . . . . . . . . . . . . . . . . . . . . . . . . . . . . . . . . . . . . 73
1.63 PGRE0177 #63 . . . . . . . . . . . . . . . . . . . . . . . . . . . . . . . . . . . . . . . 74
1.64 PGRE0177 #64 . . . . . . . . . . . . . . . . . . . . . . . . . . . . . . . . . . . . . . . 75
1.65 PGRE0177 #65 . . . . . . . . . . . . . . . . . . . . . . . . . . . . . . . . . . . . . . . 76
1.66 PGRE0177 #66 . . . . . . . . . . . . . . . . . . . . . . . . . . . . . . . . . . . . . . . 77
1.67 PGRE0177 #67 . . . . . . . . . . . . . . . . . . . . . . . . . . . . . . . . . . . . . . . 78
1.68 PGRE0177 #68 . . . . . . . . . . . . . . . . . . . . . . . . . . . . . . . . . . . . . . . 79
1.69 PGRE0177 #69 . . . . . . . . . . . . . . . . . . . . . . . . . . . . . . . . . . . . . . . 80
1.70 PGRE0177 #70 . . . . . . . . . . . . . . . . . . . . . . . . . . . . . . . . . . . . . . . 81
1.71 PGRE0177 #71 . . . . . . . . . . . . . . . . . . . . . . . . . . . . . . . . . . . . . . . 82
1.72 PGRE0177 #72 . . . . . . . . . . . . . . . . . . . . . . . . . . . . . . . . . . . . . . . 83
1.73 PGRE0177 #73 . . . . . . . . . . . . . . . . . . . . . . . . . . . . . . . . . . . . . . . 84
1.74 PGRE0177 #74 . . . . . . . . . . . . . . . . . . . . . . . . . . . . . . . . . . . . . . . 85
1.75 PGRE0177 #75 . . . . . . . . . . . . . . . . . . . . . . . . . . . . . . . . . . . . . . . 86
1.76 PGRE0177 #76 . . . . . . . . . . . . . . . . . . . . . . . . . . . . . . . . . . . . . . . 87
1.77 PGRE0177 #77 . . . . . . . . . . . . . . . . . . . . . . . . . . . . . . . . . . . . . . . 88
1.78 PGRE0177 #78 . . . . . . . . . . . . . . . . . . . . . . . . . . . . . . . . . . . . . . . 89
1.79 PGRE0177 #79 . . . . . . . . . . . . . . . . . . . . . . . . . . . . . . . . . . . . . . . 90
1.80 PGRE0177 #80 . . . . . . . . . . . . . . . . . . . . . . . . . . . . . . . . . . . . . . . 91
1.81 PGRE0177 #81 . . . . . . . . . . . . . . . . . . . . . . . . . . . . . . . . . . . . . . . 92
1.82 PGRE0177 #82 . . . . . . . . . . . . . . . . . . . . . . . . . . . . . . . . . . . . . . . 93
1.83 PGRE0177 #83 . . . . . . . . . . . . . . . . . . . . . . . . . . . . . . . . . . . . . . . 94
1.84 PGRE0177 #84 . . . . . . . . . . . . . . . . . . . . . . . . . . . . . . . . . . . . . . . 96

4
CONTENTS CONTENTS

1.85 PGRE0177 #85 . . . . . . . . . . . . . . . . . . . . . . . . . . . . . . . . . . . . . . . 97


1.86 PGRE0177 #86 . . . . . . . . . . . . . . . . . . . . . . . . . . . . . . . . . . . . . . . 98
1.87 PGRE0177 #87 . . . . . . . . . . . . . . . . . . . . . . . . . . . . . . . . . . . . . . . 100
1.88 PGRE0177 #88 . . . . . . . . . . . . . . . . . . . . . . . . . . . . . . . . . . . . . . . 101
1.89 PGRE0177 #89 . . . . . . . . . . . . . . . . . . . . . . . . . . . . . . . . . . . . . . . 102
1.90 PGRE0177 #90 . . . . . . . . . . . . . . . . . . . . . . . . . . . . . . . . . . . . . . . 104
1.91 PGRE0177 #91 . . . . . . . . . . . . . . . . . . . . . . . . . . . . . . . . . . . . . . . 105
1.92 PGRE0177 #92 . . . . . . . . . . . . . . . . . . . . . . . . . . . . . . . . . . . . . . . 107
1.93 PGRE0177 #93 . . . . . . . . . . . . . . . . . . . . . . . . . . . . . . . . . . . . . . . 109
1.94 PGRE0177 #94 . . . . . . . . . . . . . . . . . . . . . . . . . . . . . . . . . . . . . . . 111
1.95 PGRE0177 #95 . . . . . . . . . . . . . . . . . . . . . . . . . . . . . . . . . . . . . . . 112
1.96 PGRE0177 #96 . . . . . . . . . . . . . . . . . . . . . . . . . . . . . . . . . . . . . . . 114
1.97 PGRE0177 #97 . . . . . . . . . . . . . . . . . . . . . . . . . . . . . . . . . . . . . . . 115
1.98 PGRE0177 #98 . . . . . . . . . . . . . . . . . . . . . . . . . . . . . . . . . . . . . . . 117
1.99 PGRE0177 #99 . . . . . . . . . . . . . . . . . . . . . . . . . . . . . . . . . . . . . . . 118
1.100PGRE0177 #100 . . . . . . . . . . . . . . . . . . . . . . . . . . . . . . . . . . . . . . 119

5
List of Figures

1.1 Electric potential at point P in relation to ring of radius R . . . . . . . . . . . . . . . 6


1.2 P-V curve comparison of an Isothermal and Adiabtic process . . . . . . . . . . . . . 9
1.3 Ray tracing diagram of a concave mirror with a longer focal length than object distance 16
1.4 Tangential and centripetal acceleration of a particle constrained to a circle . . . . . . 29
1.5 Relativistic space time diagram for time-like, space-like and light-like reference frames 42
1.6 3 generic forms of cubic crystals . . . . . . . . . . . . . . . . . . . . . . . . . . . . . . 63

2
Chapter 1

Physics GRE Solutions

1.1 PGRE0177 #1
Recommended Solution
For a moving pendulum in which we can neglect gravity, the only two forces on the the pendulum
bob is centripetal acceleration due to its rotation and acceleration due to gravity, g. At point c,
the centripetal acceleration should be pointing upwards and acceleration due to gravity must be
pointing downwards. Sum these two forces to see that the net force on the pendulum bob can only
point up or down, and we can eliminate (B) and (D). Next, check (A) to see that the net forces at
point a and b should only have a horizontal acceleration in the left direction and, alternatively, a
horizontal acceleration in the right direction for points d and e. Since this is not true in (A), we
can eliminate it. Finally, recall that the centripetal acceleration for a pendulum will be minimized
at its peaks, so the net acceleration should be pointing downward more than left or right, which
more closely matches (C) than (E).

Correct Answer
(C)

3
1.2. PGRE0177 #2 CHAPTER 1. PHYSICS GRE SOLUTIONS

1.2 PGRE0177 #2
Recommended Solution
Start with the equation for friction,

f = µFN (1.1)
the normal force will be equal and opposite to the force due to the gravity, making Equation
1.1

f = µmg (1.2)
but since the net force in the horizontal will be rotational, f in 1.2 is then

4
1.2. PGRE0177 #2 CHAPTER 1. PHYSICS GRE SOLUTIONS

f = µmg (1.3)
mv 2
= µmg (1.4)
r
v2
= µg (1.5)
r
v2
r = (1.6)
µg
Now, convert velocity into rotational units, by

v = rω (1.7)
rev rad 1 min
   
= r 33.3 2π (1.8)
min rev 60 s
rad
≈ πr (1.9)
s
Finally, substitute Equation 1.9 into Equation 1.6 and solve to get

π2 r2
r = (1.10)
µg
µg
= (1.11)
π2
3
≈ (1.12)
π2
1
≈ (1.13)
3
which is closest to (D).

Correct Answer
(D)

5
1.3. PGRE0177 #3 CHAPTER 1. PHYSICS GRE SOLUTIONS

1.3 PGRE0177 #3

Figure 1.1: Electric potential at point P in relation to ring of radius R

Recommended Solution
Recall Kepler’s third law,
”The square of the orbital period of a planet is directly proportional to the cube of the semi-
major axis of its orbit”
or, equivalently

T 2 ∝ R3 (1.14)
3/2
T ∝ R (1.15)

which is option (D).

Correct Answer
(D)

6
1.4. PGRE0177 #4 CHAPTER 1. PHYSICS GRE SOLUTIONS

1.4 PGRE0177 #4

Recommended Solution
Despite the change in energy, we can utilize the conservation of momentum to get the final and
initial momentums

pi = 2mvi (1.16)
pf = 3mvf (1.17)
by conservation, pi = pf and we get

2mvi = 3mvf (1.18)


2
vf = vi (1.19)
3
Now, we need to find the initial and final energy of the system, as

1
Ei = (2m)vi2 = mvi2 (1.20)
2
 2
1 3 2 2
Ef = (3m)vf2 = m vi2 = mvi2 (1.21)
2 2 3 3
At which it is clear that the difference between final and initial kinetic energy is 1/3.
Correct Answer
(C)

7
1.5. PGRE0177 #5 CHAPTER 1. PHYSICS GRE SOLUTIONS

1.5 PGRE0177 #5

Recommended Solution
From the equipartion theorem, we know that the average energy for an ”ideal gas” with only
translational degrees of freedom is
3
hHi = kB T (1.22)
2
where the 3 in our 3/2 comes from the 3 degrees of freedom for translational motion. For a
harmonic oscillator, we must add to this 2 rotational degrees of freedom and 1 degree of freedom
for its single dimension of oscillations. This brings the grand total to 6 degrees of freedom, making
the average total energy
6
hHi = kB T = 3kB T (1.23)
2

Correct Answer
(D)

8
1.6. PGRE0177 #6 CHAPTER 1. PHYSICS GRE SOLUTIONS

1.6 PGRE0177 #6

Recommended Solution
The quickest solution to this problem is simply to be familiar with the P-V curves for adiabatic
and isothermal processes. You will likely have seen these in a thermodynamics lab course,

Figure 1.2: P-V curve comparison of an Isothermal and Adiabtic process

which is clearly (E).

Correct Answer
(E)

9
1.7. PGRE0177 #7 CHAPTER 1. PHYSICS GRE SOLUTIONS

1.7 PGRE0177 #7

Recommended Solution
From the diagram, we know that the two magnets have similar poles next to one another so we
shouldnt have magnetic field lines from one to another, like in (A) (C) and (D). Next, we should
get some repelling between the field lines which doesnt show up in (E), so we must select (B).

Correct Answer
(B)

10
1.8. PGRE0177 #8 CHAPTER 1. PHYSICS GRE SOLUTIONS

1.8 PGRE0177 #8
Recommended Solution
This specific problem is commonly used as one of the more simplistic, but by no means trivial,
problems to introduce the method of image charges. Going through the entire proof during this
exam would be a much more lengthy process than we can finish in a reasonable amount of time so
we can really only solve this problem by knowing that a point charge will induce an exactly equal
and opposite charge in the grounded plate, in fact choice (D).

Correct Answer
(D)

11
1.9. PGRE0177 #9 CHAPTER 1. PHYSICS GRE SOLUTIONS

1.9 PGRE0177 #9

Recommended Solution
With 5 charges set symmetrically and equidistant about the center, we should have 5 equal electric
fields canceling out fields pointing in the opposite direction. Because of symmetry, the sum of all
electric fields will equal 0.

Correct Answer
(A)

12
1.10. PGRE0177 #10 CHAPTER 1. PHYSICS GRE SOLUTIONS

1.10 PGRE0177 #10

Recommended Solution
Given two capacitors, C1 = 3 microfarad and C2 = 6 microfarad and potential difference V =
300 volt, the total energy can be found with
1
U = Ceq V 2 (1.24)
2
for capacitors in series, the equivalent capacitance is

C1 C2
Ceq = (1.25)
C1 + C2
(3 µF)(6 µF)
 
= (1.26)
(3 µF) + (6 µF)
18 µF2
= (1.27)
9 µF
= 2 µF (1.28)

plug Equation 1.28 into Equation 1.24 and solve

1
U = (2 µF)(300 volt)2 (1.29)
2
= 0.09 (1.30)

Correct Answer
(A)

13
1.11. PGRE0177 #11 CHAPTER 1. PHYSICS GRE SOLUTIONS

1.11 PGRE0177 #11

Recommended Solution
We start with the thin lens equation
1 1 1
+ = (1.31)
do di f
plug in the given values for do and f1 to solve for di

1 1 1
+ = (1.32)
40 cm di 20 cm
40 cm
1+ = 2 (1.33)
di
di = 40 cm (1.34)

since the first image is to the right of the second lens by 10 cm, for our next calculation we
must use an object distance of d0 = −10 cm. Again, using the same equation as before, we get

1 1 1
+ = (1.35)
−10 cm di 10 cm
−10 cm
1+ = −1 (1.36)
di
di = 5 cm (1.37)

which predicts a final image 5.0 cm to the right of the final lens.

Correct Answer
(A)

14
1.12. PGRE0177 #12 CHAPTER 1. PHYSICS GRE SOLUTIONS

1.12 PGRE0177 #12

Recommended Solution
Start with the mirror equation

1 1 1
+ = (1.38)
d0 di f
1 1 1
= − ?? (1.39)
di f d0
Now, since we know from the image that the focal distance is larger than the object distance, we
know that the RHS of Equation ?? must be negative and, therefore, the image distance is virtual
and behind the mirror.
Correct Answer
(E)

Alternate Solution
If we add an object at point O and do some ray tracing, we get Figure 1.3

15
1.12. PGRE0177 #12 CHAPTER 1. PHYSICS GRE SOLUTIONS

Figure 1.3: Ray tracing diagram of a concave mirror with a longer focal length than object distance

Correct Answer
(E)

16
1.13. PGRE0177 #13 CHAPTER 1. PHYSICS GRE SOLUTIONS

1.13 PGRE0177 #13

Recommended Solution
The Rayleigh Criterion can be used to give us the minimum resolution detail of a telescope,

λ
θ = 1.22 (1.40)
d
so we can solve for d in Equation 1 and plug in our known values to get

λ
d = 1.22 (1.41)
θ
600 nm
= 1.22 (1.42)
3 × 10−5 rad
= 2.5 cm (1.43)

Correct Answer
(B)

17
1.14. PGRE0177 #14 CHAPTER 1. PHYSICS GRE SOLUTIONS

1.14 PGRE0177 #14

Recommended Solution
From the description, we know that the detector is 100% efficient because exactly 50% of the
samples are being detected when pressed up exactly on one of its two sides. Since this is the case,
we just need to find out how much fewer gamma rays will hit the detector at a distance of 1 m
away. Since the rays will disperse spherically, we want to take the ratio of the samples as it passes
through a circular surface area (AC = πr2 ) to that of the surface area of a sphere (AS = 4πr2 ) at
a distance of 1 m = 100 cm. The radius of the circular surface area is 4, so we let AC = 16π cm2 .
Next, we can find the spherical surface area as

AS = 4πr2 (1.44)
2
= 4π(100 cm) (1.45)
2
= 40, 000 cm (1.46)

Finally, take the ratio of the two areas to get

AC 16π cm2
= (1.47)
AS 40, 000 cm2
= 4 × 10−4 (1.48)

Correct Answer
(C)

18
1.15. PGRE0177 #15 CHAPTER 1. PHYSICS GRE SOLUTIONS

1.15 PGRE0177 #15

Recommended Solution
Recall that precision is distinct from accuracy in that accuracy describes how close to the correct
or true value a measurement is, while precision is a measurement of how closely grouped or how
well a result can be reproduced. In other words, a group of measurements can be entirely incorrect
but still be ”precise” if they are all extremely similar to one another. Of the plots given, (A)
demonstrates the closest grouping of data points.

19
1.15. PGRE0177 #15 CHAPTER 1. PHYSICS GRE SOLUTIONS

Correct Answer
(A)

20
1.16. PGRE0177 #16 CHAPTER 1. PHYSICS GRE SOLUTIONS

1.16 PGRE0177 #16

Recommended Solution
From a laboratory methods course, you probably did some problems on measurement uncertainty
and came up with the uncertainty equation
σ
u= √ (1.49)
N
where σ is the standard deviation and N is the number of samples. Since our data set is discrete
and completed over time, we can find the standard deviation via the Poisson distribution

σ= x̄ (1.50)
where the average
√ of our current data set appears to be, x̄ = 2. This makes our standard
deviation, σ = 2. Since we want to be within 1% of the average, we take 1% of 2 to get u = 0.02.

√ 2
σ
N = (1.51)
u2
2
= (1.52)
0.022
= 5000 (1.53)

Correct Answer
(D)

21
1.17. PGRE0177 #17 CHAPTER 1. PHYSICS GRE SOLUTIONS

1.17 PGRE0177 #17

Recommended Solution
Your first approach to this problem should be to ensure that each of the possible solutions contains
all 15 electrons for phosophorous. Checking this, by summing the subscripts, you’ll find 15 for
each. This means that every single solution, except for the correct one, should have an incorrect
progression from our standard energy level diagram.

From the energy level diagram above, it should be clear that the progression is

1s2 2s2 2p6 3s2 3p3

Correct Answer
(B)

22
1.18. PGRE0177 #18 CHAPTER 1. PHYSICS GRE SOLUTIONS

1.18 PGRE0177 #18

Recommended Solution
In the problem, 79.0 eV is given as the ionization energy for both electrons which also tells us
that the ionization for each electron individually must sum to this number. We also know that the
first electron will be easier to pull from the atom because although it feels the same pull from the
positively charged nucleus, it also has a negatively charged electron trying to push it away. Thus,
we know that the first electron will have a lower ionization energy than half of the total. Since
24.6 eV is the only choice that meets this criteria, we choose (A).

Correct Answer
(A)

23
1.19. PGRE0177 #19 CHAPTER 1. PHYSICS GRE SOLUTIONS

1.19 PGRE0177 #19

Recommended Solution
The sun is powered by nuclear fusion of hydrogen atoms into helium atoms. Because the atomic
mass of hydrogen is approximately 1 and the atomic mass of helium is roughly 4, it must be true,
by conservation of energy, that 4 hydrogen atoms combine to make 1 helium atom.

Correct Answer
(B)

24
1.20. PGRE0177 #20 CHAPTER 1. PHYSICS GRE SOLUTIONS

1.20 PGRE0177 #20

Recommended Solution
Bremsstrahlung radiation refers to E&M radiation generated when a charged particle gets acceler-
ated as the result of a collision with another charged particle, i.e choice (E). This is one of those
problems that you either know, or you don’t. Unless you speak German, in which case you could
break the Bremsstrahlung into its components, bremsen ”to brake” and Strahlung ”radiation”.

Correct Answer
(E)

25
1.21. PGRE0177 #21 CHAPTER 1. PHYSICS GRE SOLUTIONS

1.21 PGRE0177 #21

Recommended Solution
The Lyman and Balmer series both refer to different types of transitions of an electron in a hydrogen
atom from one radial quantum level (n) to another. The Lyman series is a description of all such
transitions from n=r to n=1, such that r ≥ 2 and is an integer. The first Lyman transition
(commonly called Lyman-α) is n=2 going to n=1, the second (Lyman-β) involves a transition of
n=3 to n=1, etc. The Balmer series, on the other hand, involves transitions from some n=s to
n=2, such that s ≥ 3 and is an integer. The longest wavelength for both series involves the smallest
transition, i.e. n=2 going to n=1 for the Lyman Series and n=3 going to n=2 for the Balmer. The
Rydberg formula can then be used to find the wavelength for each of the two transitions
!
1 1 1
=R 2 − 2 (1.54)
λ nf ni
For this problem we won’t need to compute anything, just compare λL and λB . Doing this for
the shortest Lyman transition gives

1 1 1
 
= R 2− 2 (1.55)
λL 1 2
1 3
= R (1.56)
λL 4
λL = 4/(3R) (1.57)

and for the Balmer transition

1 1 1
 
= R 2− 2 (1.58)
λB 2 3
1 5
= R (1.59)
λB 36
λB = 36/(5R) (1.60)

26
1.21. PGRE0177 #21 CHAPTER 1. PHYSICS GRE SOLUTIONS

making the ratio

4/(3R)
λL /λB = = 5/27 (1.61)
36/(5R)

Correct Answer
(B)

27
1.22. PGRE0177 #22 CHAPTER 1. PHYSICS GRE SOLUTIONS

1.22 PGRE0177 #22

Recommended Solution
Recall our good friend Galileo, who demonstrated that an object in free fall in a vacuum will fall at
the same acceleration regardless of its mass. In the problem given, the moon is our object, space is
our vacuum and we conclude that the mass of the moon is unknowable with the given information.

Correct Answer
(B)

28
1.23. PGRE0177 #23 CHAPTER 1. PHYSICS GRE SOLUTIONS

1.23 PGRE0177 #23

Recommended Solution
In this problem, we have three vectors to consider. The first is the centripetal acceleration vector
generated by the rotation of the particle. The second two are both tangential to the circle but one
is a velocity vector, v = 10 m/s, and the other is a tangential acceleration, aT = 10 m/s2 . First,
get the net acceleration by adding aT and aC

Figure 1.4: Tangential and centripetal acceleration of a particle constrained to a circle

Since aT and aC are both 10 m/s2 , the angle between them must be 45◦ . This tells us that the
net acceleration is always 45◦ from any tangential vectors and since the velocity vector is also a
tangential vector, the angle between v and anet is 45◦ .

Correct Answer
(C)

29
1.24. PGRE0177 #24 CHAPTER 1. PHYSICS GRE SOLUTIONS

1.24 PGRE0177 #24

Recommended Solution
Start by realizing that without air resistance, the horizontal velocity will always be constant and
positive, so vx vs. t must be plot II and we can eliminate (A) and (E). Next, in the y-direction we
know that velocity must not be constant as it reaches some peak and its velocity becomes zero,
before then changing direction and increasing its velocity in a negative direction. This description
perfectly describes plot III and so we choose (C).

Correct Answer
(C)

30
1.25. PGRE0177 #25 CHAPTER 1. PHYSICS GRE SOLUTIONS

1.25 PGRE0177 #25

Recommended Solution
Start with the moment of inertia for a disk, I = 1/2mr2 , which is given in the front of your test
booklet. Next, we can find the moment of inertia for the rest of the pennies using the parallel axis
theorem,

I = Ic om + ml2 (1.62)
again, the moment of inertia for a single penny is I = 1/2mr2 but they have a radius from the
center of mass of l = 2r, so we get

1 2
Iop = mr + ml2 (1.63)
2
1 2
= mr + m(2r)2 (1.64)
2
9 2
= mr (1.65)
2
however, since we have 6 pennies, the sum of all 6 is just
54 2
Iop,6 = mr (1.66)
2
then add the moment of inertia of all 6 pennies with the inner penny to get,

31
1.25. PGRE0177 #25 CHAPTER 1. PHYSICS GRE SOLUTIONS

54 2 1 2 55 2
Iip + Iop,6 = mr + mr = mr (1.67)
2 2 2

Correct Answer
(E)

32
1.26. PGRE0177 #26 CHAPTER 1. PHYSICS GRE SOLUTIONS

1.26 PGRE0177 #26

Recommended Solution
Before the rod has moved, it only has potential energy of

mgL
U= (1.68)
2
where we use L/2 because that is the location of the center of mass. Next, recall the equation
for rotational kinetic energy, which all of the energy is converted to at the bottom of the rods fall,

mgL 1
= Iω 2 (1.69)
2 2
The moment of inertia for a rod rotating at its tip is
1
I = mL2 (1.70)
3
substitute Equation 1.70 into Equation 1.69 and substitute ω = v/r to get

mgL Iω 2
= (1.71)
2 2

33
1.26. PGRE0177 #26 CHAPTER 1. PHYSICS GRE SOLUTIONS

mL2 v 2
= (1.72)
6L2
v2
gL = (1.73)
3
p
v = 3gL (1.74)

Correct Answer
(C)

34
1.27. PGRE0177 #27 CHAPTER 1. PHYSICS GRE SOLUTIONS

1.27 PGRE0177 #27

Recommended Solution
The Hermitian operator is defined as
Z
hAi = ψ ∗ (r)Âψ(r)dr (1.75)

where  is the expectation value and is an observable. Therefore,  is, like all other observables,
is real valued and we choose (A).

Correct Answer
(A)

35
1.28. PGRE0177 #28 CHAPTER 1. PHYSICS GRE SOLUTIONS

1.28 PGRE0177 #28

Recommended Solution
Recall the condition for orthogonality,

hx|xi = 1 (1.76)
from this, we can solve for x by taking

hψ1 |ψ2 i = 0 (1.77)


(5 · 1) + (−3 · −5) + (2 · x) = 0 (1.78)
20 + 2x = 0 (1.79)
x = −10 (1.80)

Correct Answer
(E)

36
1.29. PGRE0177 #29 CHAPTER 1. PHYSICS GRE SOLUTIONS

1.29 PGRE0177 #29

Recommended Solution
Recall that the expectation value of any state is,
X
hAiψ = aj |hψ|φj i|2 (1.81)
j

where aj is the eigen value. Thus, we square each term, multiply it by its eigenvalue and sum
everything up to get

1 1 2
hÔi = − + + (1.82)
6 2 3
= 1 (1.83)

Correct Answer
(C)

37
1.30. PGRE0177 #30 CHAPTER 1. PHYSICS GRE SOLUTIONS

1.30 PGRE0177 #30

Recommended Solution
Start by realizing that the wave function should decrease as the radius goes off to infinity, which
is not true of option II and so we eliminate (B), (C) and (E). Next, as the radius goes to zero, the
wave function should go to A, which is only true of I and we can choose (A).
Note that even if you don’t recognize that the wave function should go to A when r → 0, you
can at least be sure that it shouldn’t blow up to infinity like it does in option III.

Correct Answer
(A)

38
1.31. PGRE0177 #31 CHAPTER 1. PHYSICS GRE SOLUTIONS

1.31 PGRE0177 #31

Recommended Solution
One exceedingly useful piece of information to memorize is that the ground state energy of positro-
nium is half that of the hydrogen atom,

E0,H −13.6 eV
E0,pos = = = −6.8 eV (1.84)
2 2
Then, using the Bohr model energy equation, substitute −13.6 eV with −6.8 eV and solve

1 1
 
E = −6.8 eV − (1.85)
n21 n22
1
 
= −6.8 eV 1 − (1.86)
9
= 6.0 eV (1.87)

Correct Answer
(A)

39
1.32. PGRE0177 #32 CHAPTER 1. PHYSICS GRE SOLUTIONS

1.32 PGRE0177 #32

Recommended Solution
Start by equating the total relativistic energy, E = γm0 c2 , to the rest energy, E = m0 c2 , in the
proportions given

Enet = γm0 c2 = 2m0 c2 (1.88)


which clearly tells us that the lorentz factor is γ = 2. Next, solve for velocity in the Lorentz
factor

1
2 = p (1.89)
1 − v 2 /c2
q
2 1 − v 2 /c2 = 1 (1.90)
3
v 2 /c2 = (1.91)
4√
3
v = c (1.92)
2
Finally, using our relativistic momentum and Lorentz factor, we can solve for p

p = γmo v (1.93)

3
= 2m0 c (1.94)
√ 2
= 3m0 c (1.95)

Correct Answer
(D)

40
1.33. PGRE0177 #33 CHAPTER 1. PHYSICS GRE SOLUTIONS

1.33 PGRE0177 #33

Recommended Solution
First, immediately eliminate (E) because a pion can’t achieve light speed. Next, notice that if you
try to apply the classical concept of velocity (i.e. ∆x/∆T ), you get a ridiculously large number so
relativity must be in full effect and (A) isn’t nearly fast enough. Now, start with our equation for
the space time interval

∆s2 = ∆r2 − c2 ∆t2 (1.96)


in the pion’s frame, its change in position is 0 so we have

∆s2π = −c2 ∆t2 (1.97)


− 2 8 2
= −(3 × 10 8) (1 × 10 ) (1.98)
= −9 m/s (1.99)
Now, in the lab frame, we get the space time interval,

∆s2lab = (30 m)2 − c2 ∆t2lab (1.100)


∆t2lab c2 = 909 (1.101)

909
∆lab = √ (1.102)
c2

= 101 × 10−8 (1.103)
Finally, using our standard velocity equation, plug in our values to get

∆Xlab
v = (1.104)
∆tlab
30 m
= √ (1.105)
101 × 10−8
≈ 2.98 × 108 (1.106)
Correct Answer
(D)

41
1.34. PGRE0177 #34 CHAPTER 1. PHYSICS GRE SOLUTIONS

1.34 PGRE0177 #34

Recommended Solution
From the the displacement 4-vector, we get 3 general types of space-time intervals: time-like,
space-like and light-like. These are typically drawn out on a space-time diagram as such

Figure 1.5: Relativistic space time diagram for time-like, space-like and light-like reference frames

each interval is characterized by the following criteria


time-like: if |∆x/∆t| < c, two events occur at the same location
space-like: if |∆x/∆t| > c, two events occur at the same time
light-like: if |∆x/∆t| = 0, two events are connected by a signal that moves at the speed of light.
Of the choices, the problem is describing a space-like interval which is (C).
Correct Answer
(C)

42
1.35. PGRE0177 #35 CHAPTER 1. PHYSICS GRE SOLUTIONS

1.35 PGRE0177 #35

Recommended Solution
According to our models for simple blackbody radiation, a blackbody’s power is proportional to
the 4th power of its temperature

P = κT 4 (1.107)
From this, the increase in power from tripling the temperature is

P = κ(3T )4 = 81κT 4 (1.108)


which is (E).

Correct Answer
(E)

43
1.36. PGRE0177 #36 CHAPTER 1. PHYSICS GRE SOLUTIONS

1.36 PGRE0177 #36

Recommended Solution
An adiabatic expansion is not the same thing as an isothermal expansion and an isothermal ex-
pansion is the only type which maintains a constant temperature. Additionally, from the ideal gas
equation we know that a change in volume should be accompanied by a change in temperature

P ∆V = nR∆t (1.109)

Correct Answer
(E)

44
1.37. PGRE0177 #37 CHAPTER 1. PHYSICS GRE SOLUTIONS

1.37 PGRE0177 #37

Recommended Solution
The quickest method to determine the thermodynamic work is to estimate the area under the P-V
curve. The curve is roughly triangular shaped with base of 2 and height of 300. Calculate the area
under the triangle as
1 1
P V = (2)(300) ≈ 300 (1.110)
2 2
this eliminates all but (B) and (D). We can then determine whether the work is negative or
positive by the direction of the process. Recall that a clockwise process represents a heat engine
and the work is positive. Alternatively, if the process is counter-clockwise, it’s a heat pump and
the work is negative. This process is counter-clockwise so we choose (D).

Correct Answer
(D)

45
1.38. PGRE0177 #38 CHAPTER 1. PHYSICS GRE SOLUTIONS

1.38 PGRE0177 #38

Recommended Solution
Amplitude is maximized in an RLC circuit when we’ve reached the resonant frequency. This can
be calculated by
1
ω=√ (1.111)
LC
plug in your values to get

1
ω = √ (1.112)
LC
1
C = (1.113)
Lω 2
1
= (1.114)
(25 millihenries)(1 × 103 rad)2
1
= (1.115)
2.5 × 10 millihenries-rad2
7

= 40 µF (1.116)
Correct Answer
(D)

46
1.39. PGRE0177 #39 CHAPTER 1. PHYSICS GRE SOLUTIONS

1.39 PGRE0177 #39

Recommended Solution
At low frequencies and low energies, a capacitor and a resistor look like an impassible gap so these
will act as a high pass filter (i.e. only high frequencies can pass) but an inductor will appear like
just another bit of wire. At high frequencies and high energies, capacitors and resistors can be
passed quite easily but passing through an inductor will generate a strong magnetic field and will
impede the flow, making this a low pass filter. Looking through the choices,
I. The inductor impedes high frequencies before it reaches the terminals so this can’t be a high
pass filter
II. The resistor allows the high energy to pass and the low frequencies will drop via the resistor.
We know this is a high pass filter.
III. The capacitor will allow high energies to pass through and low frequency voltage will drop.
We know this is a high pass filter.
IV. The high energy passes the resistor and passes with the capacitor without a drop in voltage.
Without a drop in voltage, this can’t be a high pass filter.

Correct Answer
(D)

47
1.40. PGRE0177 #40 CHAPTER 1. PHYSICS GRE SOLUTIONS

1.40 PGRE0177 #40

Recommended Solution
After the switch is closed, we will start off with the maximum voltage and continually decrease as
the resistor and inductor eat away at the initial voltage. From this, we can eliminate all options
but (D) and (E). Next, compare the amount of time between the two remaining options to see
that only half of the voltage dropping after 200 seconds is a vastly unrealistic voltage drop for any
circuit, so we choose (D).

Correct Answer
(D)

48
1.41. PGRE0177 #41 CHAPTER 1. PHYSICS GRE SOLUTIONS

1.41 PGRE0177 #41

Recommended Solution
The existence of magnetic charge would be, in essence, the same thing as saying that magnetic
monopoles exist. If this were the case, then II would not have to be changed to allow magnetic field
lines to diverge completely and we can eliminate (A), (C) and (D). Next, consider that a magnetic
monopole will also allow magnetic field lines to exist without curling back to its opposite pole, so
III must change.

Correct Answer
(E)

49
1.42. PGRE0177 #42 CHAPTER 1. PHYSICS GRE SOLUTIONS

1.42 PGRE0177 #42

Recommended Solution
From Lenz’s law, we know that anytime a current is generated by a change in the magnetic or electric
field, the generated emf will be such that it opposes this change in direction and magnitude. As the
center ring moves towards ring A, this increases the field present and ring A will oppose this change
with a current of opposite current, i.e. clockwise. Ring B, on the other hand, will have its field
decreased so a current will be induced to oppose this decrease, which requires an anti-clockwise
motion.

Correct Answer
(C)

50
1.43. PGRE0177 #43 CHAPTER 1. PHYSICS GRE SOLUTIONS

1.43 PGRE0177 #43

Recommended Solution
Start with the commutator relation

[AB, C] = A[B, C] + [A, C]B (1.117)


and apply it to the given commutator

[Lx Ly , Lz ] = Lx [Ly , Lz ] + [Lx , Lz ]Ly (1.118)


and replace the bracketed portions on the RHS with the commutation relations given in the
problem (while recalling that the have the commutation relation in the opposite order from left to
right simply changes the sign), to get

[Lx Ly , Lz ] = Lx [Ly , Lz ] + [Lx , Lz ]Ly (1.119)


= Lx (ih̄Lx ) + (−ih̄Ly )Ly (1.120)
= ih̄(L2x − L2y ) (1.121)

Correct Answer
(D)

51
1.44. PGRE0177 #44 CHAPTER 1. PHYSICS GRE SOLUTIONS

1.44 PGRE0177 #44

Recommended Solution
The problem gives us the energy equation as

n2 π 2 h̄2
En = (1.122)
2mL2
this tells us that the only difference between, say E1 and E2 will be the change in Energy from
the squared quantum number, n = 1, 2, 3, . . .. So, we know that all allowed energies will be some
squared integer multiple of E1 . Of those given, only (D) has a coefficient with a perfect square
value (i.e. 32 = 9).

Correct Answer
(D)

52
1.45. PGRE0177 #45 CHAPTER 1. PHYSICS GRE SOLUTIONS

1.45 PGRE0177 #45

Recommended Solution
First, recall that the expectation value for any operator is the sum of its terms, each one individually
squared and multiplied by it’s respective eigenvalue. First, find the ”ket” values for |1i, |2i and 3i,

3
|1i = h̄ω (1.123)
2
5
|2i = h̄ω (1.124)
2
7
|3i = h̄ω (1.125)
2
then multiply these by their respective coefficients, having been squared, to get

1 2 3
|ψi = √ |1i − √ |2i + √ |3i (1.126)
14 14 14
1 3 4 5 9 7
        
= h̄ω + h̄ω + h̄ω (1.127)
14 2 14 2 14 2
86
= h̄ω (1.128)
28
43
= h̄ω (1.129)
14
Correct Answer
(B)

53
1.46. PGRE0177 #46 CHAPTER 1. PHYSICS GRE SOLUTIONS

1.46 PGRE0177 #46

Recommended Solution
Start with the de Broglie wavelength equation

h
λ= (1.130)
p
and then recall the Schrdinger 1-D energy equation for a particle in a potential

p2
E= + V (x) (1.131)
2m
without doing much work, you can see that combining Equation 1.130 with Equation 1.131 will
force you to have an inverse square root to substitute the p2 in Equation 2 into the p in Equation
1. This only matches option (E) so it must be the correct choice.

Correct Answer
(E)

54
1.47. PGRE0177 #47 CHAPTER 1. PHYSICS GRE SOLUTIONS

1.47 PGRE0177 #47

Recommended Solution
First, eliminate choices (D) and (E) as both predict a negative change in entropy, which should not
be true of a gas expanding outward to occupy a new open space. Next, recall from thermodynamics
that entropy is defined as the natural log of the total number of states of a system

S = kb ln(Ω) (1.132)
where Ω is the number of states. For a volume divided into two parts, for n particles the number
of states will be

Ω = 2n (1.133)
Plugging Equation 1.133 into Equation 1.132 and applying the rules of logarithms gives

S = kb ln(Ω) (1.134)
n
= kb ln (2 ) (1.135)
= nkb ln(2) (1.136)

which is given by choice (B).

Correct Answer
(B)

55
1.48. PGRE0177 #48 CHAPTER 1. PHYSICS GRE SOLUTIONS

1.48 PGRE0177 #48

Recommended Solution
You can initially eliminate all solutions that are less than 1, as we would expect the smaller and
lighter N2 molecules to move faster than the larger and heavier O2 molecules. This step eliminates
(A), (B), and (E). Between (C) and (D), you can either take a guess, or recall Graham’s law of
effusion, which states
s
Rate1 M2
= (1.137)
Rate2 M1
at which point, (C) becomes the obvious choice.

Correct Answer
(C)

56
1.49. PGRE0177 #49 CHAPTER 1. PHYSICS GRE SOLUTIONS

1.49 PGRE0177 #49

Recommended Solution
A Maxwell-Boltzmann system has the canonical partition function

g e−Es /kb T
X
Z= (1.138)
s

Where g is the degeneracy. Since we have two energies, we will need at least two terms in our
energy equation so we can eliminate (A), (B) and (C). Next, plug everything in to see our coefficient
of 2 appear, as in (E).

g e−Es /kb T
X
Z = (1.139)
s
= 2e−/kT + 2e−2/kT (1.140)
h i
= 2 e−/kT + e−2/kT (1.141)

Correct Answer
(E)

57
1.50. PGRE0177 #50 CHAPTER 1. PHYSICS GRE SOLUTIONS

1.50 PGRE0177 #50

Recommended Solution
Recall our equation relating frequency to wavelength,
v
ν= (1.142)
λ
if our velocity is 3

vf = 0.97vo (1.143)
so the initial velocity is

ν0
νo = (1.144)
λ
v0
440 hz = (1.145)
λ
v0 = (440 hz)(λ) (1.146)

then, compare this to the final velocity

vf
νf = (1.147)
λ
(0.97v0 )
= (1.148)
λ
(0.97)(440 hz)
= (1.149)
λ
427 hz
= (1.150)
λ

Correct Answer
(B)

58
1.50. PGRE0177 #50 CHAPTER 1. PHYSICS GRE SOLUTIONS

Note: if you have any experience with wind instruments, which fortunately I do, you know
from experience that a cold instrument always starts off a bit flat and the pitch of your instrument
continually increases as it warms up. From this little fact, you should at least be able to determine
that (D) and (E) can’t be correct.

59
1.51. PGRE0177 #51 CHAPTER 1. PHYSICS GRE SOLUTIONS

1.51 PGRE0177 #51

Recommended Solution
Consider a beam of unpolarized light headed towards you and you have three ideal polarizers in
hand.

In the figure above, grey vectors indicate individual directions of oscillation, black vectors
indicate the net polarization, greyed out areas indicate areas of absorbed polarization and white
areas indicate unaffected polarization. If we wanted to completely eliminate all light using two
of the polarizers, we could place them in series with a rotation of exactly π/2 between them, for
example using the horizontal and vertical polarization. In this arrangement, anything that survived
through the horizontal polarizer would be caught by the vertical polarizer and no light would be
transmitted on the other side. However, imagine we were to place another polarizer between the
horizontal and vertical polarizers such that this third polarizer is rotated π/4 or 45 with respect to
the other two. As the light first passes through the horizontal polarizer, only half of the photons that
hit the polarizer will pass through. These photons will continue to the 45 polarizer where, again,
half of the remaining photons get absorbed and the other half pass through. After the photons
pass through the 45 polarizer, the remaining photons will spread out from -22.5 to +67.5. This

60
1.51. PGRE0177 #51 CHAPTER 1. PHYSICS GRE SOLUTIONS

occurs because linearly polarized light will always completely fill a full 90 of angular spread, which
I didn’t mention previously because the previous instances came out with a 90 spread. Finally, the
remaining photons will pass through the vertical polarizer giving a final total of 1/8 the original
number of photons

Correct Answer
(B)

61
1.52. PGRE0177 #52 CHAPTER 1. PHYSICS GRE SOLUTIONS

1.52 PGRE0177 #52

Recommended Solution
Recall the three generic types of cubic crystals: Simple Cubic, Body Centered Cubic and Face
Centered Cubic.
Sum up the total area in each cubic to find

Simple Cubic 1 Atom

Body Centered Cubic 2 Atoms

Face Centered Cubic 4 Atoms

Thus the primitive unit cell (simple cubic) contains half as much area, a3 /2

Correct Answer
(C)

62
1.53. PGRE0177 #53 CHAPTER 1. PHYSICS GRE SOLUTIONS

(a) Simple Cubic (b) Body Centered Cubic (c) Face Centered Cubic

Figure 1.6: 3 generic forms of cubic crystals

1.53 PGRE0177 #53

63
1.53. PGRE0177 #53 CHAPTER 1. PHYSICS GRE SOLUTIONS

Recommended Solution
Recall that semi-conductors, unlike regular conductors, conduct well at high temperatures and
effectively not at all at extremely low temperatures. From this, we know that the resistivity at
0 should be very high and it should decrease as temperature increases. Only (B) matches this
description.

Correct Answer
(B)

64
1.54. PGRE0177 #54 CHAPTER 1. PHYSICS GRE SOLUTIONS

1.54 PGRE0177 #54

Recommended Solution
Recall that impulse is
Z t2
I= F dt (1.151)
t1

So we just want to find the area under this plot. If you don’t recall this equation, take note
that we are given a y-axis in newtons and an x-axis in seconds. The only way to get these two
units to end with kg · m/s is to take the area. Using the equation for the area under a triangle,
you can quickly get I = 2 kg · m/s

Correct Answer
(C)

65
1.55. PGRE0177 #55 CHAPTER 1. PHYSICS GRE SOLUTIONS

1.55 PGRE0177 #55

Recommended Solution
We can quickly eliminate choices (B) and (D) based on the fact that they both suggest an asym-
metrical set of values for an exceptionally symmetric problem. We also know that (A) can’t be
right because it would violate conservation of momentum for a particle of mass m and velocity
v0 to generate two particles with mass m and velocity v0 . Finally, since we know that horizontal
momentum must be conserved and each piece gets half the horizontal momentum of the initial,
then the addition of a vertical component of velocity requires, by the Pythagorean theorem, that
the net velocity be greater than just the horizontal component. In other words

2
vnet = vx2 + vy2 (1.152)

Correct Answer
(E)

66
1.56. PGRE0177 #56 CHAPTER 1. PHYSICS GRE SOLUTIONS

1.56 PGRE0177 #56

Recommended Solution
Start by finding the difference between the air density and the helium density to get the average
density the balloon will experience

ρavg = ρair − ρHe = 1.29 kg/m3 − 0.18 kg/m3 = 1.11 kg/m3 (1.153)
Now, note that the only way to get units of m3 from kg and kg/m3 is to do the following

m
V = (1.154)
ρavg
300 kg
= (1.155)
1.11 kg/m3
= 270 m3 (1.156)

Correct Answer
(D)

67
1.57. PGRE0177 #57 CHAPTER 1. PHYSICS GRE SOLUTIONS

1.57 PGRE0177 #57

Recommended Solution
Consider that if the density of the water were to increase, so to would the force and we don’t see
this feature in (D) and (E). Next, eliminate (C) because the acceleration due to gravity,g, should
not be a part of these calculations. Finally, compare units on (A) and (B) to get

(A) ρv 2 A = (kg/m3 )(m2 /s2 )(m2 ) = kg m/s2 = N

(B) ρvA/2 = (kg/m3 )(m/s)(m2 ) = kg/s

and we choose (A).

Correct Answer
(A)

68
1.58. PGRE0177 #58 CHAPTER 1. PHYSICS GRE SOLUTIONS

1.58 PGRE0177 #58

Recommended Solution
The problem tells us that the electric field is pushing the proton in the +x-direction and, from
Fleming’s left hand rule, we can quickly deduce that the magnetic field is pushing the proton in
the -x-direction. Since we are told the proton isn’t deflected with a potential difference of V , we
know that the forces must have balanced out exactly. On the second pass, however, the potential
difference is doubled and, from the Lorentz force F = qvB, this will increase the force from the
magnetic field but not for the electric field. In the second pass, the magnetic field force will be
greater and will exceed the electric field force and the overall direction of deflection will be in the
-x-direction.

Correct Answer
(B)

69
1.59. PGRE0177 #59 CHAPTER 1. PHYSICS GRE SOLUTIONS

1.59 PGRE0177 #59

Recommended Solution
The equation for a simple harmonic oscillator is

mẍ + kx = 0 (1.157)
which we get when L = m, C = 1/k and Q = x.

Correct Answer
(B)

Alternate Solution
Start by eliminating any choices that suggest that Q = m, i.e. (C) and (E), as a change in mass
over time is not a necessary condition for a simple harmonic oscillator. Next, since Q has to be
x, from our previous elimination, we can reasonably conclude that 1/C should be 1/k so that the
second term gives us Hooke’s law, i.e. kx, and you can eliminate (A) and (D).

Correct Answer
(B)

70
1.60. PGRE0177 #60 CHAPTER 1. PHYSICS GRE SOLUTIONS

1.60 PGRE0177 #60

Recommended Solution
Start by recognizing that the flux through the Gaussian surface should have some dependence on
x, because if x blows up to infinity, so to should the flux. With (A) and (B) eliminated, next
eliminate (C) because the area under consideration should be the area cut out by the entire circle
minus the area cut out by x, not the area of the difference of the two. Finally, note that if x = 0,
the area under consideration should just be that of a circle, which is πR2 as opposed to 2πR2 .

Correct Answer
(D)

71
1.61. PGRE0177 #61 CHAPTER 1. PHYSICS GRE SOLUTIONS

1.61 PGRE0177 #61

Recommended Solution
Considering the electric field first, when an electric field orthogonal to a conductor interacts with
that conductor, the field (in a sense) disperses over it and the E field just to the left and to the
right will be 0, from which we can eliminate (B), (D) and (E). Next, as the E ~ interacts with the
~
conductor, charges are moved around and this induces a magnetic field, so B 6= 0 and we choose
(C).

Correct Answer
(C)

72
1.62. PGRE0177 #62 CHAPTER 1. PHYSICS GRE SOLUTIONS

1.62 PGRE0177 #62

Recommended Solution
Start with our cyclotron frequency equation

Bq
f= (1.158)
2πm
and then re-write Equation 1.158 to solve for m

Bq
m= (1.159)
2πf
Now, plugging everything in gives us

(π/4 teslas)(2e− )
m = (1.160)
2π(1600 hz)
e−
= (1.161)
(4)(1600 hz)
1.6 × 10−19 kg/s
= (1.162)
(4)(1600 hz)
1 × 10−22 kg
= (1.163)
4
= 2.5 × 10−23 kg (1.164)

Correct Answer
(A)

73
1.63. PGRE0177 #63 CHAPTER 1. PHYSICS GRE SOLUTIONS

1.63 PGRE0177 #63

Recommended Solution
Ideally, we would be able to recall Wien’s displacement law

b
λ= (1.165)
T
in which b is the Wien displacement law constant. Alternatively, you could quickly get exactly
the same relationship with a bit of dimensional analysis by noticing that the only way to get a
final unit of Kelvins would be to take the quotient of Wien’s displacement law constant with the
maximum wavelength. Either way you do it, convert the peak wavelength of 2 µm to 2 × 10−6 m
and solve

2.9 × 10−3 m · K
T = (1.166)
2 × 10−6 m
= 1, 500 K (1.167)

Correct Answer
(D)

74
1.64. PGRE0177 #64 CHAPTER 1. PHYSICS GRE SOLUTIONS

1.64 PGRE0177 #64

Recommended Solution
Options (C) and (D) are very much true and are frequently mentioned in a lot of pop physics
books, television and other forms of media, so these two should be easily dismissed. (E) is also true
because band spectrum, i.e. spectral lines so close together as to form a band, can’t appear when
a single atom produces a single spectral line. Lastly, (B) is true because absorption spectroscopy
and emission spectroscopy are essentially different ways of measuring the same thing, specifically
wavelengths. Only (A) is left so that is our answer.

Correct Answer
(A)

75
1.65. PGRE0177 #65 CHAPTER 1. PHYSICS GRE SOLUTIONS

1.65 PGRE0177 #65

Recommended Solution
Start with the Taylor Series expansion of ex to get an approximation

ex ≈ 1 + x (1.168)
hν/kT
e = 1 + hν/kT (1.169)

Plug this approximation into the denominator of Einstein’s formula

2
hν ehν/kT

C = 3kNA (1.170)
kT (1 + hν/kT − 1)2
2
hν ehν/kT

= 3kNA (1.171)
kT (hν/kT )2
= 3kNA ehν/kT (1.172)

Finally, as temperature blows up to infinity, ehν/kT = 1 and so we are left with

C = 3kNA (1.173)

Correct Answer
(D)

76
1.66. PGRE0177 #66 CHAPTER 1. PHYSICS GRE SOLUTIONS

1.66 PGRE0177 #66

Recommended Solution
Ignore the fact that this problem mentions anything about physics and treat it like a standard rate
problem. If Jimmy can eat a whole apple pie in 24 minutes and Susie can eat a whole apple pie in
36 minutes, how long will it take to eat that same pie if both Jimmy and Susie are sharing a single
pie?
24 ∗ 36
= 14.4 minutes (1.174)
24 + 36

Correct Answer
(D)

77
1.67. PGRE0177 #67 CHAPTER 1. PHYSICS GRE SOLUTIONS

1.67 PGRE0177 #67

Recommended Solution
First, eliminate (A) because we know that Uranium-238 can fission spontaneously. Next, eliminate
(B) because, aside from it being a very vague claim that is not typical of a GRE answer, it is a
strange conclusion to try and draw from the splitting of a Uranium atom that barely loses any
mass/energy in the process. We can then eliminate (C) simply because it isn’t true that a nuclei
with roughly half the particles of Uranium-238 would be equally massive. Finally, between (D) and
(E), recall that binding energy generally increases as we move towards the most stable element,
iron, and decreases as we move away. A change from Uranium-238 to a nuclei with A=120 would
be a movement closer to iron, so we would likely so a binding energy of 8.5 MeV/nucleon rather
than 6.7 MeV/nucleon.

Correct Answer
(E)

78
1.68. PGRE0177 #68 CHAPTER 1. PHYSICS GRE SOLUTIONS

1.68 PGRE0177 #68

Recommended Solution
Since an alpha particle is a helium atom, specifically He2+ with two protons and two neutrons,
if lithium lost an alpha particle it would have lost one to many protons to become beryllium and
(A) is eliminated. Losing an electron, positron or neutron wouldn’t alter the actual type of atom,
so emitting these wouldn’t change anything and we can eliminate (B), (C) and (D). This leaves us
(E).

Correct Answer
(E)

79
1.69. PGRE0177 #69 CHAPTER 1. PHYSICS GRE SOLUTIONS

1.69 PGRE0177 #69

Recommended Solution
From the description, we know that the light first interacts with the oil surface and the phase of
the blue light is shifted 180◦ . Next, the light interacts with the glass surface and, because of the
previous phase shift, we use the thin film interference equation

2nd = mλ (1.175)
where m = 1, 2, 3, . . .. The thinnest oil film will be when m = 1, so we solve for d in Equation
1.175 and plug in our values


d = (1.176)
2n
480 nm
= (1.177)
2(1.2)
= 200 nm (1.178)

Correct Answer
(B)

80
1.70. PGRE0177 #70 CHAPTER 1. PHYSICS GRE SOLUTIONS

1.70 PGRE0177 #70

Recommended Solution
Starting with the double slit equation

ω sin(θ) = mλ (1.179)
re-write Equation 1.179 to solve for the angle between fringes, θ, to get


sin(θ) = (1.180)
ω
from Equation 1.180, we can easily see that doubling the frequency, ω, will result in a separation
angle half as large as the initial 1.0 millimeter and we pick (B).

Correct Answer
(B)

81
1.71. PGRE0177 #71 CHAPTER 1. PHYSICS GRE SOLUTIONS

1.71 PGRE0177 #71

Recommended Solution
We can eliminate (E) right away because the proposed velocity is faster than light. Next, eliminate
(A) and (B) because we know, from the Doppler effect, that an increase in wavelength corresponds
to an object moving away from the observer. Lastly, we can choose between (C) and (D) by utilizing
the doppler redshift equation

s
λ 1 + v/c
= (1.181)
λ0 1 − v/c
s
607.5 nm 1 + v/c
= (1.182)
121.5 nm 1 − v/c
1 + v/c
25 = (1.183)
1 − v/c
24 − 26(v/c) = 0 (1.184)
v
= 12/13 (1.185)
c
v = (12/13)c (1.186)
8
v = 2.8 × 10 m/s (1.187)

Correct Answer
(D)

82
1.72. PGRE0177 #72 CHAPTER 1. PHYSICS GRE SOLUTIONS

1.72 PGRE0177 #72

Recommended Solution
Right as the string breaks the block will be accelerating due to gravity and it will also be accelerating
from the spring pulling down on the top block from the bottom block. Based on this, we know
that the net acceleration of the top block must be larger than just that from acceleration, and we
eliminate (A), (B), and (C). Lastly, summing the vertical forces on the top block, we get

− ma = −mg − k∆x (1.188)


2mg = k∆x (1.189)
which tells us that the acceleration should be 2g.
Correct Answer
(E)

83
1.73. PGRE0177 #73 CHAPTER 1. PHYSICS GRE SOLUTIONS

1.73 PGRE0177 #73

Recommended Solution
In order for block B to not fall, the gravitational force downward must be in equilibrium with the
frictional force upwards. Adding up the forces and setting the net force to 0, we get

Fnet = f − FG (1.190)
0 = µFN − mg (1.191)
mg
FN = (1.192)
µ
(20 kg)(10 m/s2 )
= (1.193)
0.50
= 400 N (1.194)

Correct Answer
(D)

84
1.74. PGRE0177 #74 CHAPTER 1. PHYSICS GRE SOLUTIONS

1.74 PGRE0177 #74

Recommended Solution
The Lagrangian equation of motion can be calculated with

d ∂L ∂L
 
− =0 (1.195)
dt ∂ q̇ ∂q
perform both derivatives to get

d ∂L
 
= 2aq̈ (1.196)
dt ∂ q̇
∂L
= 4bq 3 (1.197)
∂ q̇
plugging our results from Equation 1.197 into Equation 1.195 gives

d ∂L ∂L
 
− = 0 (1.198)
dt ∂ q̇ ∂q
2aq̈ − 4bq 3 = 0 (1.199)
3
2aq̈ = 4bq (1.200)
2b 3
q̈ = q (1.201)
a
Correct Answer
(D)

85
1.75. PGRE0177 #75 CHAPTER 1. PHYSICS GRE SOLUTIONS

1.75 PGRE0177 #75

Recommended Solution
We could approach this problem theoretically, but it will be quicker to just give the matrix a vector
and see where it goes. to simplify things, use the vector

~v = h1, 0, 0i (1.202)
multiplying everything out, we get a transformed vector

v~0 = h1/2, 3/2, 0i (1.203)
which represents a 60◦ rotation clockwise about the z-axis.

Correct Answer
(E)

86
1.76. PGRE0177 #76 CHAPTER 1. PHYSICS GRE SOLUTIONS

1.76 PGRE0177 #76

Recommended Solution
The first of the five choices we can eliminate is (D), because electrons travel quite slowly in metals,
making them exceptionally non-relativistic. We can also quickly eliminate (B) because there is no
condition specified in the problem that would take the metal out of equilibrium. Next, eliminate
(A) because electrons in a metal move freely but are generally more constrained in their degrees of
freedom than free atoms. Finally, between (C) and (E), choose (C) because interactions between
phonons and electrons are more technical than we would likely ever see on the GRE.

Correct Answer
(C)

87
1.77. PGRE0177 #77 CHAPTER 1. PHYSICS GRE SOLUTIONS

1.77 PGRE0177 #77

Recommended Solution
By Maxwell-Boltzmann statistics, we get the equation

gi e−i /kT
Ni = N (1.204)
Z
kT and the energy are both given so plug them both into Equation 1.204 and solve to get

gi e−i /kT
Ni = N (1.205)
Z
gi e(−0.1 eV)/(0.025 eV)
= N (1.206)
Z
gi e−4
= N (1.207)
Z
which results in our ratio of e−4 .

Correct Answer
(E)

88
1.78. PGRE0177 #78 CHAPTER 1. PHYSICS GRE SOLUTIONS

1.78 PGRE0177 #78

Recommended Solution
Since we are talking about changing the arrangement of neutrinos and anti-neutrinos, both of which
are leptons, we would be wise to check lepton number first. Doing so, we find the typical leptonic
decay as

µ− → e − + ν̂e + νe
L: 1 = 1 − 1 + 1
Le : 0 = 1 − 1 +0
Lµ : 1 = 0 + 0 + 1

removing the anti-neutrino from our typical lepton decays results in

µ− → e − + νe
L: 1 = 1 + 1
Le : 0 = 1 + 0
Lµ : 1 = 0 + 1

so lepton number is not conserved.

Correct Answer
(E)

89
1.79. PGRE0177 #79 CHAPTER 1. PHYSICS GRE SOLUTIONS

1.79 PGRE0177 #79

Recommended Solution
Start with the relativistic energy equation

E 2 − (pc2 ) = (mc2 )2 (1.208)


we are given E = 10 GeV and p = 8 GeV/c so we plug these in and solve for m

(10 GeV)2 − (8 GeV/c)2 c2 = m2 c4 (1.209)


2 2 2 4
(100 GeV ) − (64 GeV ) = m c (1.210)
2 2 4
36 GeV = m c (1.211)
2 2 4
m = 36 GeV /c (1.212)
2
m = 6 Gev/c (1.213)

Correct Answer
(D)

90
1.80. PGRE0177 #80 CHAPTER 1. PHYSICS GRE SOLUTIONS

1.80 PGRE0177 #80

Recommended Solution
Start by eliminating option (E) because light will move slower than c when interacting with some
medium, relativistic or not. Next, consider that when the tube and water are moving at 1/2c and
moving in the same direction as the light, we would expect the light to pass through more quickly
than if the tube was stationary. Recalling our equation for light speed in a medium,
c 3
v= = c (1.214)
n 4
so our answer should be larger than this and we eliminate (A) and (B). Finally, we can decide
between (C) and (D) by considering the sum of our two relativistic velocities using

u+v
v0 = (1.215)
1 + vu
c2
1/2c + 3/4c
= (1.216)
1 + (1/2c)(3/4c)
c2
5/4
= c (1.217)
11/8
10
= c (1.218)
11

Correct Answer
(D)

91
1.81. PGRE0177 #81 CHAPTER 1. PHYSICS GRE SOLUTIONS

1.81 PGRE0177 #81

Recommended Solution
Start with the equations for L2 and Lz

L2 Ylm (θ, φ) = l(l + 1)h̄2 Ylm (θ, φ) (1.219)


Lz Ylm (θ, φ) = mh̄Ylm (θ, φ) (1.220)

applying both eigenvalues to their respective operators, we get

l(l + 1)h̄2 = 6h̄2 (1.221)


l = 2 (1.222)

and

mh̄ = −h̄ (1.223)


m = −1 (1.224)

at which point we plug in our values for m and l into the original momentum eigenfunction and
get

Ylm (θ, φ) = Y2−1 (θ, φ) (1.225)

Correct Answer
(B)

92
1.82. PGRE0177 #82 CHAPTER 1. PHYSICS GRE SOLUTIONS

1.82 PGRE0177 #82

Recommended Solution
For a two electron system, there are three triplet state configurations and one singlet state config-
uration. The single singlet configuration occurs when the total spin, S = S1 + S2 equals 0, which
is only true when S1 = −S2 and can be written as
1
√ (|αi1 |βi2 − |αi2 |βi1 ) (1.226)
2
we can then eliminate all choices with II as an option and, more to the point, all other config-
urations must be one of the three possible triplet configurations.

Correct Answer
(D)

93
1.83. PGRE0177 #83 CHAPTER 1. PHYSICS GRE SOLUTIONS

1.83 PGRE0177 #83

Recommended Solution
Depending on how well you recall your linear algebra, you might quickly recognize that the Pauli
spin matrix given in this problem is a relatively typical transformation/rotation matrix. To demon-
strate this, you can pick any arbitrary up and down spin vectors (I’ll be using 1,2,3 & 4 just for the
purposes of distinguishing separate elements of the vectors). Start with spin up and down vectors

| ↑ i = (1, 2) (1.227)
| ↓ i = (3, 4) (1.228)

Now multiply each of those to the transformation matrix given in the problem to get the
transformed vectors

| ↑ iσx = (2, 1) (1.229)


| ↓ iσx = (4, 3) (1.230)

94
1.83. PGRE0177 #83 CHAPTER 1. PHYSICS GRE SOLUTIONS

which shows that this Pauli matrix performs an orthogonal transformation on the vectors. Of
the potential solutions, only (C) gives us the necessary result that swapping the order of the vectors
will also swap the sign.

Correct Answer
(C)

95
1.84. PGRE0177 #84 CHAPTER 1. PHYSICS GRE SOLUTIONS

1.84 PGRE0177 #84

Recommended Solution
For a single electron transition, utilize our selection rules for the orbital and total angular quantum
numbers

∆l = ±1 (1.231)
∆j = 0, ±1 (1.232)

we can eliminate transition A because ∆l = 0. For B and C, however, we have transitions

B: ∆l = −1 and ∆j = −1

C: ∆l = −1 and ∆j = 0

both of which are allowed and so we choose (D).

Correct Answer
(D)

96
1.85. PGRE0177 #85 CHAPTER 1. PHYSICS GRE SOLUTIONS

1.85 PGRE0177 #85

Recommended Solution
The resistance of the wire is related to its length and area by

ρl
R= (1.233)
a
this gives us two resistances

ρ2L
R1 = (1.234)
A
ρL
R2 = (1.235)
2A
which gives us a ratio for the resistances of R1 /R2 = 4/1. This ratio also represents, from
V = IR, the ratio of voltages

V1 4
= (1.236)
V2 1
Finally, find the net voltage as

V1 + V2 = 7 volts (1.237)
and use Equation 1.236 and Equation 1.237 to get

V = 1 + IR (1.238)
= 1 V + 1.4 V (1.239)
= 2.4 V (1.240)
Correct Answer
(A)

97
1.86. PGRE0177 #86 CHAPTER 1. PHYSICS GRE SOLUTIONS

1.86 PGRE0177 #86

Recommended Solution
Start with Ohm’s law and solve for current

V
I= (1.241)
R
Then, we can find the induced emf from a changing magnetic field by

dφB
ε = N| | (1.242)
Z dt
φB = B~ · dA
~ (1.243)

98
1.86. PGRE0177 #86 CHAPTER 1. PHYSICS GRE SOLUTIONS

The area and, therefore, the magnetic flux changes during rotation so we find the magnetic flux
as

φB = Bπr2 sin(ωt) (1.244)


and, therefore, the induced EMF is

ε = N Bπr2 ω cos(ωt) (1.245)


Finally, substitute the EMF from Equation 1.245 into Equation 1.241 to get the final solution

N Bπr2 ω cos(ωt)
I = (1.246)
R
(15 turns)(0.5 tesla)(0.01 m)2 (300 rad/sec) cos(ωt)
= (1.247)
9Ω
= 25π cos(ωt) (1.248)

Correct Answer
(E)

99
1.87. PGRE0177 #87 CHAPTER 1. PHYSICS GRE SOLUTIONS

1.87 PGRE0177 #87

Recommended Solution
For the left side of the diagram, the test charge falls inside the sphere meaning that the potential
is constant and the field is 0. This means that the only force on the test charge will be the result
of sphere Q pushing q to the left. Using Gauss’s law with a distance of 10d − d/2 = 19/2d, the net
force is

1 qQ
F = (1.249)
4π0 r2
4qQ
= (1.250)
4π0 (361d2 )
qQ
= (1.251)
361π0 d2

Correct Answer
(A)

100
1.88. PGRE0177 #88 CHAPTER 1. PHYSICS GRE SOLUTIONS

1.88 PGRE0177 #88

Recommended Solution
Eliminate (A) since a changing field must generate a magnetic field. Next, compare the units for
each of the potential solutions to get
(B): Akg
·s 2
= tesla
(C): Akg
·s 2
= tesla
(D): A·kg
s2 m 6= tesla
kg
(E): A·s2 m 6= tesla
We are left with (B) and (C) at which point you can guess or try to recall that the Biot-Savart
law is proportional to 1/4π rather than 1/4π 2

µ0 I dl × r̂
Z
B= (1.252)
4π |r|2

Correct Answer
(C)

101
1.89. PGRE0177 #89 CHAPTER 1. PHYSICS GRE SOLUTIONS

1.89 PGRE0177 #89

Recommended Solution
From conservation of angular momentum,

Lf = L0 (1.253)
When the child is in the middle of the merry-go-round, he/she/it won’t contribute to the angular
momentum, so the final momentum will be
!
M R2
Lf = Iωf = ωf (1.254)
2
however, for the initial momentum we must use both the child at a distance R and the merry-
go-round,

102
1.89. PGRE0177 #89 CHAPTER 1. PHYSICS GRE SOLUTIONS

!
M R2
Li = Iω0 = + mR2 ω0 (1.255)
2
Plug Equations 1.254 and 1.255 into Equation 1.253 to get

! !
M R2 M R2
ωf = + mR2 ω0 (1.256)
2 2
2m
ωf = ω0 + ω0 (1.257)
M
2
= 2.0 rad/sec + (2.0 rad/sec) (1.258)
5
= 2.8 rad/sec (1.259)

Correct Answer
(E)

103
1.90. PGRE0177 #90 CHAPTER 1. PHYSICS GRE SOLUTIONS

1.90 PGRE0177 #90

Recommended Solution
From simple common sense, with a shorter length in which oscillations can occur and a stronger
spring constant in Figure 1, the period should be less than that of Figure 2. From this, we can
eliminate all options that are 1 or greater, i.e. (C), (D) or (E). Between (A) and (B), recall that
the period of a SHO can be written as
r
m
T = 2π (1.260)
k
and
s
l
T = 2π (1.261)
g

which tells us that doubling k and doubling l will both scale the period by 2. However, since
each of these changes are happening in such a way as to give Figure 1 a longer period, the two
scalings combine to give a total change in period of twice as much for figure 1 or, equivalently,

T1 1
= (1.262)
T2 2

Correct Answer
(A)

104
1.91. PGRE0177 #91 CHAPTER 1. PHYSICS GRE SOLUTIONS

1.91 PGRE0177 #91

Recommended Solution
At the top of the wedge, net energy is just the gravitational energy,

Enet = mgh (1.263)


all of this energy is converted to translational and rotational at the bottom of the wedge,
1 1
mgh = mv 2 + Iω 2 (1.264)
2 2
solving for I in Equation 1.264 gives,

2mghR2
I= − R2 m (1.265)
v2
and substituting the value given for v 2

105
1.91. PGRE0177 #91 CHAPTER 1. PHYSICS GRE SOLUTIONS

2R2 mgh
I = − R2 m (1.266)
v2
7 2
= R m − R2 m (1.267)
4
3 2
= R m (1.268)
4

Correct Answer
(B)

106
1.92. PGRE0177 #92 CHAPTER 1. PHYSICS GRE SOLUTIONS

1.92 PGRE0177 #92

Recommended Solution
The Hamiltonian is the sum of kinetic and potential energy terms, as opposed to the Lagrangian
which is the difference. The energy of a harmonic oscillator is

107
1.92. PGRE0177 #92 CHAPTER 1. PHYSICS GRE SOLUTIONS

1
V = k(∆l)2 (1.269)
2
and the kinetic energy term is

1 p2
T = mv 2 = (1.270)
2 2m
adding the two of these gives us

H = T +V (1.271)
p21 p2 1
= + 2 + k(l − l0 )2 (1.272)
2m " 2m 2 #
1 p1 p22
2
2
= + + k(l − l0 ) (1.273)
2 m m

Correct Answer
(E)

108
1.93. PGRE0177 #93 CHAPTER 1. PHYSICS GRE SOLUTIONS

1.93 PGRE0177 #93

Recommended Solution
If you do well with your physics history, you may recall that Bohr radius is defined as the smallest
possible orbital distance for the hydrogen atom in its ground state, which is sufficient to choose a0 .

Correct Answer
(C)

Alternate Solution
If you don’t recall the fact in the recommended solution, start by taking the squared wavefunction
and multiplying it by a spherical shell

" #2
1 −r/a0
dP = √ 3/2
e 4πr2 dr (1.274)
πa0
4 2 −2r/a0
= r e dr (1.275)
a30

differentiating dP in Equation 1.275 with respect to r and setting it equal to 0 allows us to


solve for the minimum radius

2 2 −2r/a0
2re−2r/a0 − r e = 0 (1.276)
a0
r
 
2re−2r/a0 1 − = 0 (1.277)
a0

109
1.93. PGRE0177 #93 CHAPTER 1. PHYSICS GRE SOLUTIONS

at which point, it is easy to solve for the radius as

r = a0 (1.278)

Correct Answer
(C)

110
1.94. PGRE0177 #94 CHAPTER 1. PHYSICS GRE SOLUTIONS

1.94 PGRE0177 #94

Recommended Solution
From quantum mechanics, the first order energy shift for our specific hamiltonian is

En(1) = hn0 |V (a + a† )|n0 i (1.279)


multiply the Hamiltonian out to get
 2
a + a† = a2 + aa† + a† a + a†2 (1.280)

at which point we can eliminate a2 and a†2 by orthogonality. Finally, solve for the energy with
our remaining terms

E = hn|(aa† + a† a|ni (1.281)


= (2n + 1)V (1.282)
= 5V (1.283)

Correct Answer
(E)

111
1.95. PGRE0177 #95 CHAPTER 1. PHYSICS GRE SOLUTIONS

1.95 PGRE0177 #95

Recommended Solution
Start by recalling the relative permitivity equation

ε(ω)
κ= (1.284)
ε0
where ε(ω) and ε0 are the absolute permitivity and electric constant, respectively. Next, re-
calling that the Electric field is inversely proportional to ε(ω), which you could potentially realize
from Coulomb’s law, we get

1
E ∝ (1.285)
ε(ω)

112
1.95. PGRE0177 #95 CHAPTER 1. PHYSICS GRE SOLUTIONS

1
∝ (1.286)
0 κ
Then applying E0 ∝ 1/ε0 , we get

E0
E= (1.287)
κ

Correct Answer
(A)

113
1.96. PGRE0177 #96 CHAPTER 1. PHYSICS GRE SOLUTIONS

1.96 PGRE0177 #96

Recommended Solution
Recall the Larmor formula, which gives the power radiated by a charged object,

e2 a2
P = (1.288)
6π0 c3
since the sphere is not moving, just expanding at a stationary location, we get an acceleration,
a = 0. Plugging this into Equation 1 clearly results in a total radiated power of zero as well.

Correct Answer
(E)

114
1.97. PGRE0177 #97 CHAPTER 1. PHYSICS GRE SOLUTIONS

1.97 PGRE0177 #97

115
1.97. PGRE0177 #97 CHAPTER 1. PHYSICS GRE SOLUTIONS

Recommended Solution
Consider the limiting case of θ = 0. In this case, there will be no difference between δθ0 and θ0 ,
which tells us that the solution should have some dependence on θ and, furthermore, that δθ0 = 0
when θ = 0. This eliminates (A), (B) and (C) due to their lack of theta dependence and (D) based
on the its failure to have δθ0 = 0 when θ = 0. Note that taking the limit in (D) requires you to
utilize L’hospitals rule to deal with the division of 0/0, at which point you get a pair of cosines
which go to 1 at θ0 = 0 and θ = 0 and keeps δθ0 from going to 0.

Correct Answer
(E)

Alternate Solution
Rather than applying a limiting case to the angle, θ, we can apply the limiting case n = 1. When
n = 1, there is no transition to a new index of refraction and δθ0 should be equivalent to θ0 . We can
immediately eliminate (D) because it lacks the dependence on n and we can next eliminate (A),
(B) and (C) because their lack of θ0 dependence means we could never get δθ0 = θ0 when n = 1.

Correct Answer
(E)

116
1.98. PGRE0177 #98 CHAPTER 1. PHYSICS GRE SOLUTIONS

1.98 PGRE0177 #98

Recommended Solution
Examining both sums in the expression, it should become quite clear that whatever it represents,
it will have units of energy. Looking at the units for the potential solutions,

(A) Average energy is an energy!

(B) The denominator of this particular expression is the partition function. You don’t really need
to know that, however, provided you recognize that it is not an energy.

(C) Absolutely not an energy.

(D) Probability of a certain energy value is not, in itself, an energy value.

(E) Entropy has units of J/K so it is not an energy.

Correct Answer
(A)

117
1.99. PGRE0177 #99 CHAPTER 1. PHYSICS GRE SOLUTIONS

1.99 PGRE0177 #99

Recommended Solution
Considering the problem qualitatively, after the collision occurs, we have three particles each with
a mass m and some non-zero velocity. This means that our final energy, which by conservation
of momentum must equal our initial energy, must be the sum of three particles with rest energy
of mc2 (i.e. net rest energy will be 3mc2 ) and some kinetic energy. This tells us that the initial
photon energy must be greater than just the final rest energies so we can eliminate (A), (B) and
(C). Finally, equating the coefficients 4 and 5 in choices (D) and (E), respectively, to the Lorentz
factor, γ, gives us a strong hint that higher values of γ will give us less realistic particle velocities
(i.e. far too high) and we can choose (D).

Correct Answer
(D)

118
1.100. PGRE0177 #100 CHAPTER 1. PHYSICS GRE SOLUTIONS

1.100 PGRE0177 #100

Recommended Solution
Recalling the acronym for the visible light spectrum, ROYGBIV, it should be clear that red light
has a longer wavelength and lower energy than green light. Red light is given to us in this instance
as λred = 632.82 nm so we know that the wavelength should be less than this and we can eliminate
(D) and (E). Next, recall width of the visible spectrum is roughly between 380 nm and 750 nm
and that green light is exactly in the middle of the spectrum. Thus, averaging the two wavelengths
should give us a good approximation for the wavelength of green light
380 nm + 750 nm 1130 nm
= = 565 nm (1.289)
2 2
and this average is closest to (B).

Correct Answer
(B)

119

You might also like